Đến nội dung

Hình ảnh

$(\frac{1}{a}+\frac{1}{c})(\frac{1}{b}+\frac{1}{d})\leq \frac{4}{\sqrt{3}}$

bất đẳng thức và cực trị

  • Please log in to reply
Chủ đề này có 2 trả lời

#1
khonggiohan

khonggiohan

    Hạ sĩ

  • Thành viên
  • 53 Bài viết

  cho a,b,c,d dương thỏa mãn $\frac{1}{2+a^{2}}+\frac{1}{2+b^{2}}+\frac{1}{2+c^{2}}+\frac{1}{2+d^{2}}=1$

 CMR $(\frac{1}{a}+\frac{1}{c})(\frac{1}{b}+\frac{1}{d})\leq \frac{4}{\sqrt{3}}$


             

                 Đời cho tôi 1 vai diễn lớn, chỉ hiềm nỗi tôi không hiểu nổi cốt truyện


#2
Hoang Tung 126

Hoang Tung 126

    Thiếu tá

  • Thành viên
  • 2061 Bài viết

  cho a,b,c,d dương thỏa mãn $\frac{1}{2+a^{2}}+\frac{1}{2+b^{2}}+\frac{1}{2+c^{2}}+\frac{1}{2+d^{2}}=1$

 CMR $(\frac{1}{a}+\frac{1}{c})(\frac{1}{b}+\frac{1}{d})\leq \frac{4}{\sqrt{3}}$

Đề bài đúng phải là :$\frac{1}{a^4+1}+\frac{1}{b^4+1}+\frac{1}{c^4+1}+\frac{1}{d^4+1}=1$.CMR:$(\frac{1}{a}+\frac{1}{c})(\frac{1}{b}+\frac{1}{d})\leq \frac{4}{\sqrt{3}}$

Đặt $\frac{1}{a}=x,\frac{1}{b}=y,\frac{1}{c}=z,\frac{1}{d}=t$.Do đó BDT cần CM:$< = > (x+z)(y+t)\leq \frac{4}{\sqrt{3}}$

Ta có:$1=\sum \frac{1}{a^4+1}=\sum \frac{1}{1+(\frac{1}{x})^4}=\sum \frac{x^4}{x^4+1}= > (1-\frac{x^4}{x^4+1})=\frac{y^4}{y^4+1}+\frac{z^4}{z^4+1}+\frac{t^4}{t^4+1}= > \frac{1}{x^4+1}\geq 3\sqrt[3]{\frac{(yzt)^4}{(y^4+1)(z^4+1)(t^4+1)}}$(Do áp dụng Cosi 3 số)

 Lập các biểu thức tương tự rồi nhân theo vế $= > \frac{1}{(x^4+1)(y^4+1)(z^4+1)(t^4+1)}\geq 3^4.\sqrt[3]{\frac{(xyzt)^4}{(x^4+1)^4(y^4+1)^4(z^4+1)^4(t^4+1)^4}}=> (xyzt)^4\leq 3^4= > xyzt\leq 3$

Theo Cauchy-Swatch có:$1=\sum \frac{x^4}{x^4+1}\geq \frac{(\sum x^2)^2}{\sum x^4+4}= > \sum x^4+4\geq (\sum x^2)^2= > 2\geq (x^2y^2+z^2t^2)+(y^2z^2+x^2t^2)+(x^2z^2+y^2t^2)\geq (x^2y^2+z^2t^2)+(y^2z^2+x^2t^2)+2xyzt=(xy+zt)^2+(yz+xt)^2-2xyzt\geq (xy+zt)^2+(yz+xt)^2-\frac{2}{3}\geq \frac{(xy+zt+yz+xt)^2}{2}-\frac{2}{3}=\frac{(x+z)^2(y+t)^2}{2}-\frac{2}{3}= > 2+\frac{2}{3}\geq \frac{(x+z)^2(y+t)^2}{2}= > (x+z)^2(y+t)^2\leq \frac{16}{3}= > (x+z)(y+t)\leq \frac{4}{\sqrt{3}}$(ĐPCM)

Dấu  =xảy ra khi $x=y=z=t=\frac{1}{\sqrt[4]{3}}< = > a=b=c=d=\sqrt[4]{3}$



#3
shinichikudo201

shinichikudo201

    Thiếu úy

  • Thành viên
  • 521 Bài viết

Đề bài đúng phải là :$\frac{1}{a^4+1}+\frac{1}{b^4+1}+\frac{1}{c^4+1}+\frac{1}{d^4+1}=1$.CMR:$(\frac{1}{a}+\frac{1}{c})(\frac{1}{b}+\frac{1}{d})\leq \frac{4}{\sqrt{3}}$

Sao bạn sửa đề khác quá thế..có lẽ đó ko phải sự nhầm lẫn đâu..............


It is the quality of one's convictions that determines successnot the number of followers






Được gắn nhãn với một hoặc nhiều trong số những từ khóa sau: bất đẳng thức và cực trị

0 người đang xem chủ đề

0 thành viên, 0 khách, 0 thành viên ẩn danh